On a multisection of a series

ceiling-and-floor-functionssequences-and-series

Let $x$ be a real number and $\lfloor{x}\rfloor$ denote the greatest integer less than or equal to $x$. Let $r$ be a positive integer. Let $a$ and $n$ be nonnegative integers such that $n-a+1\geq r$. Prove that

$$ \sum_{k=a}^{n} f(k) = \sum_{j=0}^{r-1} \sum_{k=\lfloor{\frac{a+r-1-j}{r}}\rfloor}^{\lfloor{\frac{n-j}{r}}\rfloor} f(rk+j).$$

I couldn't arrive at any proof.

Also it is noted that for a fixed $r$, this identity partitions the sum modulo remainder classes. The number of remainder classes is indexed by $j$. Could you explain this note?

I generally have trouble with series whose indices involves greatest integer function, are there any fast techniques to solve them?

Best Answer

Let $m$ be an integer, note that there exists a $k\in\mathbb{N}$ and a $0\leq j\leq r-1$ such that $m=rk+j$, where $j$ is the remainder of $m$ divided by $r$. We also say that $m$ is $j$ modulo $r$. In the sum above what happens is we fix $j$ and then run through all $k$ such that $a\leq rk+j\leq n$. This gives the sum modulo a remainder class. Adding all the remainder classes then gives you the total sum.

To right down the identity all you have to figure out are the correct bounds for $k$. So let $0\leq j\leq r-1$. We have to solve $$a\leq rk+j\leq n\Leftrightarrow \frac{a-j}{r}\leq k\leq \frac{n-j}{r}$$ So the upper bound of $k$ if $\lfloor\frac{n-j}{r}\rfloor$, i.e. the biggest integer smaller than \frac{n-j}{r}. With the same reasoning the lower bound is given by $\lceil\frac{a-j}{r}\rceil$, however as they require the lower bound it requires a bit more thinking.

Now suppose $a=sr+t$ with $s\in\mathbb{N}$ and $0\leq t\leq r-1$, then $$\lceil\frac{a-j}{r}\rceil=\left\lceil\frac{sr+t-j}{r}\right\rceil=\begin{cases}s&\text{ if }t-j\leq0\\s+1&\text{ if }t-j>0\end{cases}.$$ However since we have to use the lower bound we have to convert this. Instinctively you might think to use $\lfloor\frac{a+r-j}{r}\rfloor$, but then we have $$\lfloor\frac{a+r-j}{r}\rfloor=\left\lfloor\frac{(s+1)r+t-j}{r}\right\rfloor=\begin{cases}s&\text{ if }t-j<0\\s+1&\text{ if }t-j\geq0\end{cases},$$ i.e. it goes wrong for $t-j=0$ when $a-j$ is an integer. Hence we use $\lfloor\frac{a+r-1-j}{r}\rfloor$ to get the desired result. (Note that for the worst case scenario $t=0$ and $j=r-1$ we have $\frac{a+r-1-j}{r}=\frac{sr+r-1-r+1}{r}=\frac{sr}{r}=s$ as required.)

So the sum of the elements of the remainder class of some $0\leq j\leq r-1$ between $a$ and $n$ is given by $$\sum_{k=\left\lfloor\frac{a+r-1-j}{r}\right\rfloor}^{\lfloor\frac{n-j}{r}\rfloor}f(rk+j)$$ and the total sum is given by the sum of the sums of remainder classes $$\sum^{n}_{i=a}f(i)=\sum^{r-1}_{j=1}\sum_{k=\left\lfloor\frac{a+r-1-j}{r}\right\rfloor}^{\lfloor\frac{n-j}{r}\rfloor}f(rk+j).$$

I hope this helps, let me know if a part is still unclear.

Related Question